I saw this problem online some time ago, and I have been trying to solve this inequality:
\(x,y,z >0\), prove \[\frac{x^4}{8x^3+5y^3}+\frac{y^4}{8y^3+5z^3}+\frac{z^4}{8z^3+5x^3} \geqslant \frac{x+y+z}{13}\]
This question was asked online years ago and no one has proved it with an "elegant" way. I decided to share it here to all who haven't seen this problem yet, as I find it interesting and exciting while solving the problem (although I haven't found an elegant proof) XD
Have fun!
Upon typing the title I remembered a quote I saw in Evan Chen's book :D :
Graders received some elegant solutions, some not-so-elegant solutions, and some so-not-elegant solutions. --MOP 2012
Easy Math Editor
This discussion board is a place to discuss our Daily Challenges and the math and science related to those challenges. Explanations are more than just a solution — they should explain the steps and thinking strategies that you used to obtain the solution. Comments should further the discussion of math and science.
When posting on Brilliant:
*italics*
or_italics_
**bold**
or__bold__
paragraph 1
paragraph 2
[example link](https://brilliant.org)
> This is a quote
\(
...\)
or\[
...\]
to ensure proper formatting.2 \times 3
2^{34}
a_{i-1}
\frac{2}{3}
\sqrt{2}
\sum_{i=1}^3
\sin \theta
\boxed{123}
Comments
There is a solution that involves using symmetric polynomials, but you will need a good computer to do this.
Log in to reply
This problem can be solved using just pen and paper.
This inequality was used as a proposal problem for National TST of an Asian country a few years back. However, upon receiving the official solution, the committee decided to drop this problem immediately. They don't believe that any students can solve this problem in 3 hour time frame.
Log in to reply
Of course you can, but a proper proof would require symmetric polynomials, for which simplifying them (in this case) would be a right piece of work for a computer, let alone a human being. Fortunately, I have a very good PC that can help me out here; many others on this website would struggle to do so.
Log in to reply
these.
Ok. I was wondering if it could be solved using inequalities/theorems likeLog in to reply
Hey,
cyclic∑8x3+5y3x4≥13x+y+z≥133(xyz)31
So, can we prove cyclic∑8x3+5y3x4≥133(xyz)31
Cause if we can, Then it is quite easy.... IN THE CASE WHEN x,y,z≥0
SOLUTION
cyclic∑8x3+5y3x4≥13x+y+z≥133(xyz)31
So We can say cyclic∑8x3+5y3x4×13(x3+y3+z3)≥(x2+y2+z2)2 By Cauchy-Schwarz Inequality more precisely Titu's Lemma
Also we can see that x2+y2+z2≥xy+yz+zx
Thus, cyclic∑8x3+5y3x4×(x3+y3+z3)≥13(xy+yz+zx)2
And, 13(xy+yz+zx)2≥139(xyz)34 By AM-GM-HM Inequality
Finally, 13(xy+yz+zx)2≥133(xyz)31×3xyz
So cyclic∑8x3+5y3x4×(x3+y3+z3)≥133(xyz)31×3xyz........................1
Also, x3+y3+z3≥3xyz.....................2
Now, as x,y,z≥0 So divide inequality 1 by inequality 2 ,
Hence the final inequality becomes cyclic∑8x3+5y3x4≥133(xyz)31
If it wasn't metioned that x,y,z≥0 we couldn't divide two inequalities directly.
Please check if it is correct.... I am in grade 10th
Log in to reply
Yes, you did prove that ∑cyc8x3+5y3x4≥133(xyz)31, but unfortunately, that doesn't quite prove the original problem statement. This is because ∑cyc8x3+5y3x4≥133(xyz)31 can definitely mean 13x+y+z≥∑cyc8x3+5y3x4≥133(xyz)31.
I'm gonna give a simple example. Suppose that you want to prove x>y. You DO know that y>z. And somehow you do manage to prove that x>z. Does it mean that x>y?
No! The conditions y>z and x>z does not imply that x>y. See: If x=2,y=3,z=1: y=3>z=1 TRUE, and x=2>z=1 TRUE, however, x=2<y=3!
Log in to reply
Yes, I agree with you.. Thank you for correcting me.
There, if you're interested in more inequalities :) https://artofproblemsolving.com/articles/files/MildorfInequalities.pdf
Log in to reply
I will surely access it.